miércoles, 19 de enero de 2011

Problema del dia: Jueves 20 de enero - ALG

Para empezar la serie de problemas de algebra, uno de desigualdades.

Muestra que para cualesquiera numeros reales positivos $x$, $y$, $z$ se tiene que

$$4(x+y+z)^3 > 27 (x^2 y+y^2 z +z^2 x).$$

16 comentarios:

Unknown dijo...
Este comentario ha sido eliminado por el autor.
Unknown dijo...

Hay dos casos, a. $x\geq y\geq z$. y b. $x\leq y\leq z$
Caso a.
Entonces digamos que $A=x-y,B=y-z$ entonces $x=A+B+z,y=B+z,z=z$ con $A,B,z\ge 0$. Pasamos todo al lado izquierdo y expandemos,entonces
$$4A^3-3A^2B+9A^2z-6AB^2+36ABz+27Az^2+5B^3$$
$$+36B^2z+54Bz^2+27z^3\ge 0$$
Y factorizando de otra forma sale
$$9z(A+2B)^2+27z^2(A+2B+z)+(4A+5B)(A-B)^2\ge 0$$
Como $A,B,z\ge 0$ entonces esa ultima desigualdad es cierta, porlo que la primera desigualdad tambien lo es.
Caso b. $x\leq y\leq z$. En particular $(x-y)(x-z)(y-z)\leq 0$ porlotanto $x^2y+y^2z+z^2x\leq xy^2+yz^2+zx^2$. Tenemos que $z\geq y\geq x$ y por el caso a tenemos que
$$4(x+y+z)^3\ge 27(z^2y+y^2x+x^2z)\ge 27(x^2y+y^2x+x^2z)$$

Unknown dijo...

Una cosa que quiero notar, para evitar que se eliminara parte de la solucion no use el simbolo del teclado de menor estricto, sino el '\ge' de latex. Pero alparecer esto es lo mismo que '\geq'. La mayoria de los $\geq$ de arriba son > y viceversa

IwakuraIsa dijo...

puedes usar \textless y \textgreater

rvaldez dijo...

Diego, parece que tu solución esta bien!!!

Unknown dijo...
Este comentario ha sido eliminado por el autor.
Unknown dijo...
Este comentario ha sido eliminado por el autor.
Unknown dijo...

Otra manera de hacerlo es con multiplicadores de lagragne. Podemos suponer sin perdida de generalidad que $x+y+z=1$
Sea $f(x,y,z)=x^2y+y^2z+z^2x$ y $g(x,y,z)=x+y+z-1$. Entonces $D=f+g\lambda$. Tenemos que las hipotesis del problema se cumplen.
El metodo de los multiplicadores de lagrange dice que el maximo de $f(x,y,z)$ sujeto a $g(x,y,z)=0$ se da cuando las deriviadas parciales de $f$ sobre $x,y,z,\lambda$ son 0. En otras palabras
$$2xy+z^2+\lambda=0$$
$$2yz+x^2+\lambda=0$$
$$2zx+y^2+\lambda=0$$
$$x+y+z-1=0$$
Restando la primera y la segunda igualdad y la segunda y la tercera quedan otras tres igualdades, ya sin $\lamda$.
$$2xy+z^2-2yz-x^2=0$$
$$2yz+x^2-2zx-y^2=0$$
factorizamos y queda
$$(z-x)(z+x-2y)=0$$
$$(x-y)(x+y-2z)=0$$
usando que $x+y+z=1$ tenemos que $z+x-2y=-(3y-1))$ porlotanto tenemos
$$(z-x)(3y-1)=0$$
$$(x-y)(3z-1)=0$$
Asi que $x-y=z-x=0,3y-1=x-y=0,z-x=3z-1=0$ ó $3y-1=3z-1=0$. Checando cada caso es facil deducir que $x=y=z=1/3$. Porlotanto $f(1/3,1/3,1/3)=1/9<4/27$ es el maximo que $f$ puede alcanzar.

Unknown dijo...

Fue extraño que el sistema de ecuaciones no fuera horriblemente feo.

Georges dijo...

Diego pero si tomas x=2/3 y=1/3 z=0 (bueno tomas el limite a esos números para asegurar que z es positivo) tienes que x+y+z=1 pero tambien tienes que f(x,y,z)=4/27 osea el maximo se alcanza en este caso, no cuando x=y=z=1/3.
O que onda???

Georges dijo...

Mi solucion esta bastante fea, aunque no me da tanta pena ya viendo las soluciones de Diego. Alguien tiene una solución bonita???

Lo unico bonito de mi solución es que probe una desigualdad mas fuerte, probe que 4(x+y+z)^3>27(x^2y+y^2z+z^2x+xyz).

Lo que hize fue suponer spg que x es el menor, entonces existen a,b positivos tales que a+x=y b+x=z.

Luego expando todo y lo paso del lado izquierdo y eso que me queda se puede factorizar como 9(a^2-ab+b^2)+(2b-a)^2(b+4a)>0, la igualdad se da cuando a=b=0 es decir cuando x=y=z. (Perdon por no poner las cuentas, pero la verdad no tiene caso)

Georges dijo...

Diego si no te convence ese ejemplo toma f(1/2,1/3,1/6)=25/216 mayor que 1/9.

Unknown dijo...

perdon, con lagrange saque el minimo, no el maximo.

Anónimo dijo...

Pues vemos que si multiplicamos x, y, z por un real positivo k, podemos factorizar la k de ambos lados y nos queda la desigualdad original, entonces podemos multiplicar por cualquier k y si es verdad para ese caso específico, también lo es para los demás.

sea k= 1/(x+y+z), demostremos que 8(x+y+z)^3 > 27 (x^2*y + x^2*z + y^2*x + y^2*z + z^2*x + z^2*y)
para x+y+z=1 (porque k= 1/(x+y+z))
y de esto veremos que como x>=y>=z o x= 27 (x^2*y + x^2*z + y^2*x + y^2*z + z^2*x + z^2*y)
8 > 27 (x^2*y + x^2*z + y^2*x + y^2*z + z^2*x + z^2*y) +9 - 9(x+y+z)^3
9(x^3 + y^3 + z^3) + 54xyz>1

Como 54xyz>0, si demostramos que 9(x^3 + y^3 + (1-x-y)^3)>=1 acabamos

9(1 + 3(x^2 + y^2 + 2xy – x – y - x^2*y - y^2*x)>=1
8/27 + (x+y) ^2 >= (x+y)(xy+1)
8/27>=(x+y)(x-1)(y-1)=(x+y)(1-x)(1-y)
2/3>=[(x+y)(x-1)(y-1)=(x+y)(1-x)(1-y)]ˆ(1/3)
Y por AM-GM es verdad y acabamos

Anónimo dijo...

Perdon, el final queda asi:

2/3>=[(x+y)(1-x)(1-y)]ˆ(1/3)
Y por AM-GM es verdad y acabamos

rvaldez dijo...

Hola David,

El problema de Algebra del dia jueves 20 de enero, solo fue comentado y resuelto por
Diego, Georges y Jorge Chuck

Publicar un comentario